Solve Parametric Equation for Motion of Particle XY

  • Thread starter Thread starter evilpostingmong
  • Start date Start date
  • Tags Tags
    Parametric
Click For Summary
The discussion focuses on solving the parametric equations x=cos(πt) and y=sin(πt) to describe the motion of a particle in the xy-plane. The original attempt incorrectly expressed the equations, leading to confusion about the particle's motion. The correct interpretation shows that the particle moves counterclockwise along a circular path, with boundaries at t=0 and t=2. The participant seeks clarification on their solution, believing it to be valid despite the initial misunderstanding. The response emphasizes correcting the equation format to achieve the correct motion description.
evilpostingmong
Messages
338
Reaction score
0

Homework Statement


Describe motion of a particle w/ position xy


Homework Equations



x=cospi(t) y=sinpi(t)

The Attempt at a Solution


solving for t
t=x/cospi
so y=sinpi(x)/cospi
y=tanpi(x)

interval=at least one at most 2
since tan(x)=0 at pi and 2pi
and this is where the boundaries are so the particle travels from pi and 2pi counterclockwise
but the textbook has a different (but reasonable) answer in that it squares cos pi and
sin pi to equal 1 and uses the circle equation but ends up with the same diagram and
direction. If I went wrong, can you please tell me where I went wrong since my answer
seems legit to me.
 
Physics news on Phys.org
x= cos\pit
should become
t=cos^{-1}x/\pi
 
have some π …

evilpostingmong said:
x=cospi(t) y=sinpi(t)

If I went wrong, can you please tell me where I went wrong since my answer
seems legit to me.

Hi evilpostingmong! :smile:

Your initial equations are wrong.

It isn't x=cosπ(t) y=sinπ(t);

it's x=cos(πt) y=sin(πt).

Now try! :smile:

(oh … and here's some π and other things to pack in your bag …)
 
Question: A clock's minute hand has length 4 and its hour hand has length 3. What is the distance between the tips at the moment when it is increasing most rapidly?(Putnam Exam Question) Answer: Making assumption that both the hands moves at constant angular velocities, the answer is ## \sqrt{7} .## But don't you think this assumption is somewhat doubtful and wrong?

Similar threads

  • · Replies 11 ·
Replies
11
Views
3K
  • · Replies 1 ·
Replies
1
Views
9K
  • · Replies 3 ·
Replies
3
Views
3K
  • · Replies 3 ·
Replies
3
Views
2K
  • · Replies 2 ·
Replies
2
Views
3K
  • · Replies 2 ·
Replies
2
Views
2K
  • · Replies 6 ·
Replies
6
Views
2K
  • · Replies 8 ·
Replies
8
Views
1K
  • · Replies 7 ·
Replies
7
Views
1K
  • · Replies 5 ·
Replies
5
Views
1K